Calculate the following line integral

Click For Summary
The integral of the vector field f(x,y) = (xy, y) along the curve γ(t) = (r cos(t), r sin(t)) from 0 to 2π evaluates to 0. Several participants confirm this result, emphasizing the need for familiarity with trigonometric identities in the calculations. One user suggests exploring alternative methods, such as finding a potential function u where ∇u = f. There is a request for verification of the calculations, particularly for those who prefer typed solutions over images. The discussion highlights the importance of understanding vector fields and line integrals in this context.
lep11
Messages
380
Reaction score
7

Homework Statement


Let ##f(x,y)=(xy,y)## and ##\gamma:[0,2\pi]\rightarrowℝ^2##,##\gamma(t)=(r\cos(t),r\sin(t))##, ##r>0##. Calculate ##\int_\gamma{f{\cdot}d\gamma}##.

Homework Equations

The Attempt at a Solution


The answer is 0. Here's my work. However, this method requires that you are familiar with some useful trig identities.
2cpuy6v.jpg


Could someone please take a look at it and check if it's correct? Are there alternative ways? I have also tried to find the potential function ##u##, ##\nabla{u}=f##...
 
Last edited:
Physics news on Phys.org
lep11 said:

Homework Statement


Let ##f(x,y)=(xy,y)## and ##\gamma:[0,2\pi]\rightarrowℝ^2##,##\gamma(t)=(r\cos(t),r\sin(t))##, ##r>0##. Calculate ##\int_\gamma{f{\cdot}d\gamma}##.

Homework Equations

The Attempt at a Solution


The answer is 0. Here's my work. However, this method requires that you are familiar with some useful trig identities.
2cpuy6v.jpg


Could someone please take a look at it and check if it's correct? Are there alternative ways? I have also tried to find the potential function ##u##, ##\nabla{u}=f##...

I get an answer of 0 as well, but I have not checked your work because I do not look at posted images, but only at typed versions.
 
Question: A clock's minute hand has length 4 and its hour hand has length 3. What is the distance between the tips at the moment when it is increasing most rapidly?(Putnam Exam Question) Answer: Making assumption that both the hands moves at constant angular velocities, the answer is ## \sqrt{7} .## But don't you think this assumption is somewhat doubtful and wrong?

Similar threads

  • · Replies 9 ·
Replies
9
Views
1K
Replies
6
Views
3K
  • · Replies 5 ·
Replies
5
Views
2K
  • · Replies 2 ·
Replies
2
Views
2K
  • · Replies 1 ·
Replies
1
Views
2K
Replies
12
Views
2K
  • · Replies 8 ·
Replies
8
Views
2K
  • · Replies 8 ·
Replies
8
Views
2K
  • · Replies 19 ·
Replies
19
Views
2K
Replies
1
Views
1K